{"id":37596,"date":"2022-02-26T18:11:27","date_gmt":"2022-02-26T12:41:27","guid":{"rendered":"https:\/\/mcq-questions.com\/?p=37596"},"modified":"2022-02-26T18:11:27","modified_gmt":"2022-02-26T12:41:27","slug":"mcq-questions-for-class-10-maths-chapter-7-coordinate-geometry","status":"publish","type":"post","link":"https:\/\/mcq-questions.com\/mcq-questions-for-class-10-maths-chapter-7-coordinate-geometry\/","title":{"rendered":"MCQ Questions for Class 10 Maths Chapter 7 Coordinate Geometry"},"content":{"rendered":"

Coordinate Geometry Class 10 MCQ Questions with Answers<\/h2>\n

Question 1.<\/p>\n

The distance of the point P (2, 3) from the x-axis is:<\/h2>\n

(A) 2
\n(B) 3
\n(C) 1
\n(D) 5
\nAnswer:
\n(B) 3<\/p>\n

Explanation:
\nSince coordinate of a given point is the distance of point from x-axis.
\n\"MCQ<\/p>\n

\"MCQ<\/p>\n

Question 2.<\/p>\n

The distance between the points A (0, 6) and B (0,-2) is:<\/h2>\n

(A) 6
\n(B) 8
\n(C) 4
\n(D) 2
\nAnswer:
\n(B) 8<\/p>\n

Explanation:
\nThe distance between two points (x1<\/sub>, y1<\/sub>) and (x2<\/sub>, y2<\/sub> ) is given as,
\nd = \\(\\sqrt{\\left(x_{2}-x_{1}\\right)^{2}+\\left(y_{2}-y_{1}\\right)^{2}}\\)
\nWhere, x1, = 0, y1 = 6 and x2 = 0, y2 = -2 So, distance between A (0, 6) and B (0, -2):
\nAB = \\(\\sqrt{(0-0)^{2}+(-2-6)^{2}}\\)
\n= \\(\\sqrt{0+(-8)^{2}}\/latex]
\n= [latex]\\sqrt{8^{2}}\\)
\n= 8<\/p>\n

Question 3.<\/p>\n

The distance of the point P (-6,8) from the origin is:<\/h2>\n

(A) 8
\n(B) 2\u221a7
\n(C) 10
\n(D) 6
\nAnswer:
\n(C) 10<\/p>\n

Explanation:
\nDistance between two points (x1<\/sub>, y1<\/sub>) and (x2<\/sub>, y2<\/sub>) is given as,
\nd = \\(\\sqrt{\\left(x_{2}-x_{1}\\right)^{2}+\\left(y_{2}-y_{1}\\right)^{2}}\\)
\nWhere, x1<\/sub>= -6, y1<\/sub> =8 and x2<\/sub> = 0, y2<\/sub> = 0 So, distance between P (-6,8) and origin O (0,0) is given by,
\nPO = \\(\\sqrt{[0-(-6)]^{2}+[(0-8)]^{2}}\\)
\n= \\(\\sqrt{(6)^{2}+(-8)^{2}}\\)
\n= \\(\\sqrt{36+64}\\)
\n= \\(\\sqrt{100}\\)
\n= 10<\/p>\n

Question 4.<\/p>\n

The distance between the points (0, 5) and (-5, 0) is:<\/h2>\n

(A) 5
\n(B) 5\u221a2
\n(C) 2\u221a5
\n(D) 10
\nAnswer:
\n(B) 5\u221a2<\/p>\n

Explanation:
\nDistance between two points (x1<\/sub>,y1<\/sub>) and (x2<\/sub>,y2<\/sub>) is given as,
\n= \\(\\sqrt{(-5-0)^{2}+(0-5)^{2}} \\)
\nWhere, x1<\/sub>= 0, y1<\/sub>= 5 and x2<\/sub> = -5, y2<\/sub> = 0 So that, distance between the point (0, 5) and I (-5,0),
\n= \\(\\sqrt{\\left(x_{2}-x_{1}\\right)^{2}+\\left(y_{2}-y_{1}\\right)^{2}}\\)
\n= \\(\\sqrt{(-5-0)^{2}+(0-5)^{2}}\\)
\n= \\(\\sqrt{25+25}\\)
\n= \\(\\sqrt{50}\\)
\n= 5\u221a2<\/p>\n

Question 5.<\/p>\n

AOBC is a rectangle whose three vertices are vertices A (0, 3), O (0, 0) and B (5, 0). The length of its diagonal is:<\/h2>\n

(A) 5
\n(B) 3
\n(C) \u221a34
\n(D) 4
\nAnswer:
\n(C) \u221a34<\/p>\n

Explanation:
\nAccording to the question, a triangle can be represented as:
\n\"MCQ
\n\u2234 Distance between the points A (0,3) and B (5, 0) is
\nAB = \\(\\sqrt{(5-0)^{2}+(0-3)^{2}}\\)
\n= \\(\\sqrt{25+9}\\)
\n= \\(\\sqrt{34}\\)
\nHence, the required length of diagonal is \u221a34.<\/p>\n

\"MCQ<\/p>\n

Question 6.<\/p>\n

The perimeter of a triangle with vertices (0, 4), (0,0) and (3,0) is:<\/h2>\n

(A) 5
\n(B) 12
\n(C) 11
\n(D)7+\u221a5
\nAnswer:
\n(B) 12
\nExplanation:
\nSince, Perimeter of triangle = OA + AB + OB
\n\"MCQ
\nHere OA = 4 units, AB = 5 units (using pythagoras theorem in triangle AOB) and OB =3 units.
\nTherefore, Perimeter of triangle = 4 + 5+3
\n= 12 units.
\nQuestion 7.<\/p>\n

The points (-4,0), (4,0) and (0,3) are the vertices of a:<\/h2>\n

(A) right triangle
\n(B) isosceles triangle
\n(C) equilateral triangle
\n(D) scalene triangle E
\nAnswer:
\n(B) isosceles triangle<\/p>\n

Explanation:
\nLet X (-4, 0), Y (4, 0) and Z (0, 3) are the given vertices.
\nNow, distance between X ( -4,0) and Y (4, 0),
\nXY = \\(\\sqrt{[4-(-4)]^{2}+(0-0)^{2}}\\)
\n= \\(\\sqrt{(4+4)^{2}}\\)
\n= \\(\\sqrt{8^{2}}\\)
\n= 8
\nDistance between Y(4, 0) and Z (0, 3),
\nYZ = \\(\\sqrt{(0-4)^{2}+(3-0)^{2}}\\)
\n= \\(\\sqrt{16+9}\\)
\n= \\(\\sqrt{25}\\)
\n= 5
\nDistance between Y (4,0) and Z (0, 3),
\nZX = \\(\\sqrt{[0-(-4)]^{2}+(3-0)^{2}}\\)
\n= \\(\\sqrt{16+9}\\)
\n= \\(\\sqrt{25}\\)
\n= 5
\nAs, YZ = ZX, i.c., two sidps of the triangle ane H equal.
\nSo that, the AXYZ is an isosceles triangle.<\/p>\n

Question 8.<\/p>\n

The coordinates of the point which is equidistant from the three vertices of the \u2206AOB as shown in the figure is:<\/h2>\n

\"MCQ
\n(A) (x, y)
\n(B) (y,x)
\n(C) \\(\\left(\\frac{x}{2}, \\frac{y}{2}\\right)\\)
\n(D) \\(\\left(\\frac{y}{2}, \\frac{x}{2}\\right)\\)
\nAnswer:
\n(A) (x, y)<\/p>\n

Explanation:
\nFrom the given figure, coordinates of three vertices of a triangle arc: O (0, 0), A (0,2y) and B (2x, 0). Suppose the required point be P whose coordinates are (h, k) Now, P is equidistant trom the three vertices of \u2206AOB, therefore,
\nPO = PA = PB
\nor (PO)2<\/sup> = (PA)2<\/sup> = (PB)2<\/sup>
\nBy using distance formula, we get:
\n\\(\\left(\\sqrt{(h-0)^{2}+(k-0)^{2}}\\right)^{2}\\) = \\(\\left(\\sqrt{(h-0)^{2}+(k-2 y)^{2}}\\right)^{2}\\)
\n= \\(\\left(\\sqrt{(h-2 x)^{2}+(5-0)^{2}}\\right)^{2}\\)
\n\u21d2 h2<\/sup> + k2<\/sup> = h2<\/sup> +(k – 2y)2 = (h – 2x)2<\/sup> + k2<\/sup> ………(i)
\n\u21d2 k2<\/sup> = k2<\/sup> + 4y2<\/sup> – 4yk
\n\u21d2 0 = 4y2<\/sup> – 4yk
\n\u21d2 4y (y – k) = 0
\n\u21d2 y = k
\nTaking first and second parts of EQuestion (i),
\nh2<\/sup> + k2<\/sup> = (h- 2x)2<\/sup> + k2<\/sup>
\n\u21d2 h2<\/sup> = h2<\/sup> + 4x2<\/sup> – 4xh
\n\u21d2 4x (x – h) = 0
\n\u21d2 h = x
\nRequired coordinates of the point are (h, k) = (x, y)<\/p>\n

Assertion and Reason Based MCQs<\/span><\/p>\n

Directions: In the following questions, A statement of Assertion (A) is followed by a statement of Reason (R). Mark the correct choice as.
\n(A) Both A and R are true and R is the correct explanation of A
\n(B) Both A and R are true but R is NOT the correct explanation of A
\n(C) A is true but R is false
\n(D) A is false and R is True<\/p>\n

Question 1.<\/p>\n

Assertion (A): If the distance between the point (4, p) and (1,0) is 5, then the value of p is 4.
\nReason (R): The point which divides the line segment joining the points (7, – 6) and (3,4) in ratio 1: 2 internally lies in the fourth quadrant.<\/h2>\n

Answer:
\n(D) A is false and R is True<\/p>\n

Explanation:
\nIn case of assertion:
\nDistance between two points (x1<\/sub>,y1<\/sub>) and (x2<\/sup>, y2<\/sup>) is given as,
\nd = \\(\\sqrt{\\left(x_{2}+x_{1}\\right)^{2}+\\left(y_{2}+y_{1}\\right)^{2}}\\)
\nwhere,
\n(x1<\/sub>,y1<\/sub>) = (4, p)
\n(x2<\/sub>,y2<\/sub>) = (1, 0)
\nAnd, d = 5
\nPut the values, we have
\n52<\/sup> = (1 \u2014 4)2<\/sup> + (0 – p)2<\/sup>
\n25 = (-3)2<\/sup> + (-p)2<\/sup>
\n25-9 = p2<\/sup>
\n16 = p2<\/sup>
\n+4, -4 = p
\n\u2234 Assertion is incorrect.
\nIn case of reason:
\nLet (x, y) be the point
\nThen, x = \\(\\frac{m x_{2}+n x_{1}}{m+n}\\) and y = \\(\\frac{m y_{2}+n y_{1}}{m+n}\\) ………..(i)
\nHere, x1<\/sub> = 7, y1<\/sub>= -6, x1<\/sub> = 3, y1<\/sub> = 4, m = 1 and n = 2
\n\u2234 x = \\(\\frac{1(3)+2(7)}{1+2}\\)
\n\u21d2 x = \\(\\frac{3+14}{3}=\\frac{17}{3}\\)
\nAnd, y = \\(\\frac{1(4)+2(-6)}{1+2}\\)
\n\u21d2 y = \\(\\frac{4-12}{3}=-\\frac{8}{3}\\)
\nSo, the required point (x,y) = \\(\\left(\\frac{17}{3},-\\frac{8}{3}\\right)\\) lies in IV th<\/sup> quadrant.
\n\u2234 Reason is correct.
\nHence, assertion is incorrect but reason is correct.<\/p>\n

\"MCQ<\/p>\n

Question 2.<\/p>\n

Assertion (A): AABC with vertices A(-2, 0), B(2, 0) and C(0,2) is similar to ADEF with vertices D(-4,0), E(4,0) and F(0,4).
\nReason (R): A circle has its centre at the origin and a point P(5, 0) lies on it. The point Q(6,8) lies outside the circle.<\/h2>\n

Answer:
\n(B) Both A and R are true but R is NOT the correct explanation of A<\/p>\n

Explanation:
\nIn case of assertion:
\nBy using Distance Formula,
\n\u2234Distance between A (-2,0) and B (2,0),
\nAB = \\(\\sqrt{[2-(-2)]^{2}+(0-0)^{2}}\\)=4
\n[\u2234distance between the points (x1, y1) and (x2,y2)
\nD = \\(\\sqrt{\\left(x_{2}-x_{1}\\right)^{2}+\\left(y_{2}-y_{1}\\right)^{2}}\\)
\nSimilarly, distance between B (2,0) and C (0, 2),
\nBC = \\(\\sqrt{(0-2)^{2}+(2-0)^{2}}\\) = \\(\\sqrt{4+4}\\) = 2\u221a2
\nIn \u2206ABC, distance between C (0, 2) and A (-2,0),
\nCA = \\(\\sqrt{\\left[0-(-2)^{2}\\right]+(2-0)^{2}}\\) = \\(\\sqrt{4+4}\\) = 2\u221a2
\n\u2234Distance between F (0,4) and D (-4, 0),
\nFD = \\(\\sqrt{(0+4)^{2}+(0-4)^{2}}\\) = \\(\\sqrt{4^{2}+(4)^{2}}\\) = 4\u221a2
\n\u2234Distance between F (o,4) and E (4,0) and D (-4, 0),
\nFE = \\(\\sqrt{(0-4)^{2}+(4-0)^{2}}\\) = \\(\\sqrt{4^{2}+4^{2}}\\) = 4\u221a2
\nand distance between E (4,0) and D (-4, 0),
\nED = \\(\\sqrt{[4-(-4)]^{2}+(0)^{2}}\\) = \\(\\sqrt{8^{2}}\\) = 4\u221a2
\nNow, \\(\\frac{A B}{O E}\\) = \\(\\frac{4}{9}\\) = \\(\\frac{1}{2}\\),
\n\\(\\frac{A C}{D E}\\) = \\(\\frac{2 \\sqrt{2}}{4 \\sqrt{2}}\\) = \\(\\frac{1}{2}\\)
\n\\(\\frac{B C}{E F}\\) = \\(\\frac{2 \\sqrt{2}}{4 \\sqrt{2}}\\)= \\(\\frac{1}{2}\\)
\n\u2234 \\(\\frac{A B}{D E}\\) = \\(\\frac{A C}{D F}\\) = \\(\\frac{B C}{E F}\\)
\nHere, we see that sides of AABC and AFDE are proportional.
\nTherefore, by SSS similarity rule, both the triangles are similar.
\n\u2234 Assertion is correct.
\nIn case of reason:
\nFaint Q (6, 8) will lie outside the circle if its distance from the centre of circle is greater than the radius of the circle.
\nDistance between centre O (0,0) and P (5,0),
\nOP = \\(\\sqrt{(5-0)^{2}+(0-0)^{2}}\\)
\n= \\(\\sqrt{5^{2}+0^{2}}\\)
\n= 5
\nAs, point P lies at the circle, therefore, OP = Radius of circle.
\nDistance between centre O (0,0) and Q (6,8),
\nOQ = \\(\\sqrt{(6-0)^{2}+(8-0)^{2}}\\)
\n= \\(\\sqrt{6^{2}+8^{2}}\\)
\n= \\(\\sqrt{36+64}\\)
\n= \\(\\sqrt{100}\\)
\n= 10
\nAs OQ > OP, therefore, point Q (6, 8) lies outside of the circle.
\n\u2234 Reason is correct.
\nHence, both assertion and reason are correct but reason is not the correct explanation for assertion.<\/p>\n

Question 3.<\/p>\n

Assertion (A): The ordinate of a point A on y-axis is 5 and B has coordinates (-3,1). Then the length of AB is 5 units.
\nReason (R): The point A(2, 7) lies on the perpendicular bisector of line segment joining the points P(6,5) and Q(0, -4).<\/h2>\n

Answer:
\n(C) A is true but R is false<\/p>\n

Explanation:
\nIn case of assertion:
\nHere, A \u2192 (0,5) and B \u2192 (-3,1)
\nAB = \\(\\sqrt{\\left(x_{2}-x_{1}\\right)^{2}+\\left(y_{2}-y_{1}\\right)^{2}}\\)
\n= \\(\\sqrt{(-3-0)^{2}+(1-5)^{2}}\\)
\n= \\(\\sqrt{9+16}\\)
\n= \\(\\sqrt{25}\\)
\n= 5 units
\n\u2234 Assertion is correct.
\nIn case of reason:
\nIf A (2, 7) lies on perpendicular bisector of P (6,5) and Q (0, -4), then
\nAP = AQ
\n\u2234 By using Distance Formula,
\nAP = \\(\\sqrt{(6-2)^{2}+(5-7)^{2}}\\)
\n= \\(\\sqrt{(4)^{2}+(-2)^{2}}\\)
\n= \\(\\sqrt{20}\\)
\nAnd AQ = \\(\\sqrt{(0-2)^{2}+(-4-7)^{2}}\\)
\n= \\(\\sqrt{(-2)^{2}+(-11)^{2}}\\)
\n= \\(\\sqrt{125}\\)
\nAs, AP AQ
\nThercfore, A does not lie on the perpendicular bisector of PQuestion
\n\u2234 Reason is incorrect.
\nHence, assertion is correct but reason is incorrect.<\/p>\n

Case – Based MCQs<\/span><\/p>\n

Attempt any four sub-parts from each question. Each sub-part carries 1 mark.
\nI. Read the following text and answer the below questions:
\nThe diagram show the plans for a sun room. It will be built onto the wall of a house. The four walls of the sun room are square clear glass panels. The roof is made using,<\/p>\n